Sei sulla pagina 1di 3

Boundedness Theorem

Recall from the Functions Bounded on a Set page that a function f is bounded on a set A if for
every M∈R, M>0, then ∀x∈A, we have that ∣f(x)∣<M.

We will now look at an important theorem known as the boundedness theorem which says that if
f is a continuous function over the closed and bounded interval I, then f is a bounded function
over the set I.

Theorem 1 (Boundedness): If I=[a,b] is a closed and bounded interval, and f:I→R is a


continuous function on I, then f is bounded on I.

 Proof: We will carry out this proof by contradiction. Let I=[a,b] be a closed and
bounded interval, and let f:I→R be a continuous function on I.

 Now suppose that f is NOT bounded on the interval I. Then for any n∈N there exists an
element xn∈I such that ∣f(xn)∣>n. Now consider the sequence (xn). Since I is a bounded
interval, this implies that the sequence (xn) which contains elements from I is also
bounded. Therefore, by The Bolzano Weierstrass Theorem there exists a subsequence
(xnk) that converges to L, that is limk→∞xnk=L.

 Now since I is also a closed interval, we have that the elements of the sequence (xnk) are
contained within I, and so we have that L∈I.
 Now since f:I→R is a continuous function at L (since L∈I and f is continuous on I, then
we have that by the Sequential Criterion for the Continuity of a Function we have that
(f(xnk)) converges to f(L), that is limk→∞f(xnk)=f(L). Since the sequence (f(xnk)) is
convergent, then we know that this sequence is also bounded.

 But this is a contradiction. Notice that ∣f(xnk)∣>nk≥k for all k∈N, and so our supposition
that f was not bounded on I was false. Therefore f is bounded on I. ■

We should make special note that the conclusion to the boundedness theorem is guaranteed to
hold provided that:

 (1) f:I→R is a continuous function on I.

 (2) I is a closed interval.

 (3) I is a bounded interval.

We will now look at an example of where the conclusion to the boundedness theorem holds
provided these three conditions are met, and some examples of where the conclusion does not
hold when some of the conditions are NOT met.

Example 1
Verify that the function f:[0,2]→R defined by f(x)=x2 is bounded.

We first note that f defined by f(x)=x2 is continuous on all of R and so f is also continuous on
the interval [0,2]. Furthermore, I=[0,2] is a closed bounded interval. Since f is an increasing
function on the interval [0,2] as f′>0 on [0,2] we conclude that ∣f(x)∣≤4 for all x∈[0,2].

Example 2
Verify that the function f:[−1,1]→R defined by f(x)=1x does not satisfy the conditions of
the boundedness theorem.

Note that f is not continuous on all of I=[−1,1]. In fact, f is not continuous at x=0 and so we
are not guaranteed that f is to be bounded on I. Precisely, f is not bounded on [−1,1], since for
all n∈N there exists 1n∈[−1,1] such that f(1n)=n, and we know that the set of natural numbers
is not bounded, that is there DOES NOT exist an M∈R, M>0 such that ∣f(1n)∣=n<M for all
n∈N.

Example 3
Verify that the function f:(0,2)→R defined by f(x)=1x does not satisfy the conditions of
the boundedness theorem.

In this example, f is continuous on all of I=(0,2), however, this interval is not closed. We can
use the same argument in example 2 to show that hence f is not bounded, or we can use limits to
show that as x→0+, then f(x)→∞.

Example 4
Verify that the function f:[0,∞)→∞ defined by f(x)=x does not satisfy the conditions of
the boundedness theorem.

In this example, f is continuous on all of [0,∞), and this interval is also closed. However, this
interval is not bounded, and using the Archimedean property we can show that f is not bounded
as a result.

Potrebbero piacerti anche